Quantcast
  • Register
PhysicsOverflow is a next-generation academic platform for physicists and astronomers, including a community peer review system and a postgraduate-level discussion forum analogous to MathOverflow.

Welcome to PhysicsOverflow! PhysicsOverflow is an open platform for community peer review and graduate-level Physics discussion.

Please help promote PhysicsOverflow ads elsewhere if you like it.

News

PO is now at the Physics Department of Bielefeld University!

New printer friendly PO pages!

Migration to Bielefeld University was successful!

Please vote for this year's PhysicsOverflow ads!

Please do help out in categorising submissions. Submit a paper to PhysicsOverflow!

... see more

Tools for paper authors

Submit paper
Claim Paper Authorship

Tools for SE users

Search User
Reclaim SE Account
Request Account Merger
Nativise imported posts
Claim post (deleted users)
Import SE post

Users whose questions have been imported from Physics Stack Exchange, Theoretical Physics Stack Exchange, or any other Stack Exchange site are kindly requested to reclaim their account and not to register as a new user.

Public \(\beta\) tools

Report a bug with a feature
Request a new functionality
404 page design
Send feedback

Attributions

(propose a free ad)

Site Statistics

205 submissions , 163 unreviewed
5,047 questions , 2,200 unanswered
5,345 answers , 22,709 comments
1,470 users with positive rep
816 active unimported users
More ...

  Bloch's theorem and Bloch's state

+ 1 like - 0 dislike
1014 views

The question is not so much about the theorem, but more about what it means in this context: see this link.

So yes, because of Bloch's theorem the Hamiltonian eigenstates in a crystalline system can be written as \begin{align} \psi_{n,\vec{k}}(\vec{r})=e^{i\vec{k}\cdot\vec{r}}u_{n,\vec{k}}(\vec{r}), \end{align} and so the Berry connection can be defined: \begin{align} A_{n}(\vec{k})=i\langle n(\vec{k})|\nabla_{\vec{k}}|n(\vec{k})\rangle, \end{align} but what in the world is $|n(\vec{k})\rangle$?

I've read a few articles on topological insulators and they always seem to start off with the Bloch wavefunction $e^{i\vec{k}\cdot\vec{r}} u_k(\vec{r})$, and then somehow they magically get the ket $|u(\vec{k})\rangle$ from which the Berry connection is defined... is $|u(\vec{k})\rangle$ the column vector comprised of the Fourier coefficients of $u_\vec{k}(\vec{r})$ w.r.t. $e^{i\vec{G}\cdot\vec{r}}$ or what?

This post imported from StackExchange Physics at 2014-03-24 04:14 (UCT), posted by SE-user nervxxx
asked Oct 19, 2013 in Theoretical Physics by nervxxx (210 points) [ no revision ]
As the article says, $n$ is the band index. If you were to represent the ket $|u_{n}(\mathbf{k})\rangle$ in a basis (in this case the basis of a Hilbert space spanned by bands) then you could write it as a column vector where each component corresponds to the Bloch wavefunction for each band (labeled by $n$). Note: for a the simplest topological insulator model you need at least two bands: valence and conduction band. In that case you'll have a $2 \times 1$ column vector.

This post imported from StackExchange Physics at 2014-03-24 04:14 (UCT), posted by SE-user NanoPhys
@NanoPhys How do you get rid of the position $\vec{r}$ dependence then? Is that absorbed in the way the inner product is defined? But $|n(\vec{k})\rangle$ should exist on its own, and should be independent of $\vec{r}$. How does one get it from $u_{n,\vec{k}}(\vec{r})$?

This post imported from StackExchange Physics at 2014-03-24 04:14 (UCT), posted by SE-user nervxxx
Yes, that is correct. The $\mathbf{r}$ does indeed get absorbed in the definition of the inner product. Note that a Bloch state is uniquely labeled by $n$ and $\mathbf{k}$ independent of what basis it is represented in. In your case, you are writing down the Bloch "wavefunction" $\psi_{n,\mathbf{k}}(\mathbf{r})\propto e^{i\mathbf{k}\cdot\mathbf{r}}u_{n,\mathbf{k}}(\mathbf{r})$ in the position basis. The inner product $\langle u_{n}(\mathbf{k})|\dots|u_{n}(\mathbf{k})\rangle$ has to be basis independent.

This post imported from StackExchange Physics at 2014-03-24 04:14 (UCT), posted by SE-user NanoPhys
@NanoPhys So what would Bloch's theorem be, without going into any explicit basis (in particular, the position basis)? What I mean is, what can we say about the abstract kets $|\psi\rangle$? See arxiv.org/pdf/1304.5693v3.pdf , eqn 27-30. I'm not sure why that is true unless he's already assuming that the kets are already in the position basis

This post imported from StackExchange Physics at 2014-03-24 04:14 (UCT), posted by SE-user nervxxx
Equation (28) is valid. The $e^{i\mathbf{k}\cdot\mathbf{r}}$ is simply a phase factor; two kets can, in general, be related in that fashion. To make better sense of it, take a $\langle\mathbf{r}|$ on both sides. You'll get $\langle\mathbf{r}|\psi_{n\mathbf{k}}\rangle = e^{i \mathbf{k} \cdot\mathbf{r}} \langle \mathbf{r}|u_{n\mathbf{k}}\rangle\Rightarrow\psi_{n\mathbf{k}}(\mathbf{r}) = e^{i\mathbf{k}\cdot\mathbf{r}}u_{n\mathbf{k}}(\mathbf{r})$, which is our familiar Bloch wavefunction

This post imported from StackExchange Physics at 2014-03-24 04:14 (UCT), posted by SE-user NanoPhys

Your answer

Please use answers only to (at least partly) answer questions. To comment, discuss, or ask for clarification, leave a comment instead.
To mask links under text, please type your text, highlight it, and click the "link" button. You can then enter your link URL.
Please consult the FAQ for as to how to format your post.
This is the answer box; if you want to write a comment instead, please use the 'add comment' button.
Live preview (may slow down editor)   Preview
Your name to display (optional):
Privacy: Your email address will only be used for sending these notifications.
Anti-spam verification:
If you are a human please identify the position of the character covered by the symbol $\varnothing$ in the following word:
p$\hbar$ysicsOverf$\varnothing$ow
Then drag the red bullet below over the corresponding character of our banner. When you drop it there, the bullet changes to green (on slow internet connections after a few seconds).
Please complete the anti-spam verification




user contributions licensed under cc by-sa 3.0 with attribution required

Your rights
...